What does the Question Stem tell us?
The question stem is slightly unusual, but this is still a Match the Flaw question. We're given a flawed argument in the stimulus, and asked to find the answer choice that contains matching flawed reasoning.
Break down the Stimulus:
"We're given two premises and a conclusion:
P1: brick house —> front yard
P2: front yard —most—> two stories
------------------------------------------------------
C: brick house —most—> two stories
It's important to recognize that only the first premise is a conditional statement. The second premise and conclusion are quantified statements. They don't guarantee anything. We can't link conditional statements with quantified statements in the same way that we can link conditional statements.
Suppose you live in one of the brick houses. The first statement tells us that you have a front yard. But there could be other, non-brick houses that also have front yards. And those non-brick houses with front yards might be the only ones that have two stories. None of the brick houses need to be among those included in the second premise. This is why the conclusion is flawed."
Any prephrase?
To be a match, the correct answer has to contain the same type of reasoning and the same flaw. We can eliminate answers choices if either the logical structure or the flaw doesn't match.
Answer choice analysis:
A) "The conclusion in this answer choice could be a match, but the first premise is reversed, so this isn't an analogous argument:
L —> P
L —most—> R
-----------------------
P —most—> R"
B) This answer choice has two quantified ("most") premises, and no conditional premise, so it can't be a match.
C) The conclusion in this answer choice is a "some" statement (some public servants have not run for office), not a "most" statement, and so is the second premise. This should make us suspicious of answer choice (C), but there's another problem. In the original argument, the sufficient condition of the conditional premise (brick house) was included in the conclusion. The sufficient condition of the conditional premise in this answer choice (legislator) does not appear in the conclusion. It's not a match.
D) "Correct. This answer choice has the same structure, and therefore the same flaw:
legislator —> pub. servant
pub. servant —most—> ~ run for office
------------------------------------------------------
legislator —most—> ~ run for office"
E) Like answer choice (B), this answer choice has two quantified ("most") premises, and no conditional premise, so it can't be a match.
Takeaway/Pattern: Matching questions are often based on conditional logic. You can often eliminate several answer choices based on obvious mismatches. More difficult questions will present two answer choices with similar logical structures. You'll need to compare them carefully to the stimulus to determine which one is correct.
#officialexplanation